- Mon Jan 20, 2014 12:00 am
#38060
Complete Question Explanation
(The complete setup for this game can be found here: lsat/viewtopic.php?t=14958)
The correct answer choice is (A)
This is the first of two Rule Substitution questions on the December 2013 LSAT. The question stem requires you to replace the first rule of the game, which created the following relationship:
The rule itself is relatively straightforward. Without it, we are left with the following sequencing relationships:
Since Additional Effects are easier to spot, look for answer choices that contain such conditions first. You should immediately notice, for instance, that answer choices (B) and (D) add conditions that are extraneous to our original rule set: K need not be released earlier than L, and F does not have to be released earlier than K. Both of these answer choices should quickly be eliminated.
Among the Partial Match answer choices, (E) is the easiest one to eliminate. While the rule we are asked to substitute requires either F or K to be released first, this inference—by itself—does not have the exact same effect on the order in which the films are released. For instance, if K were released first, the relationship between F, J, and L would remain completely uncertain. Because answer choice (E) can limit K without limiting F in any manner, it is incorrect.
Next, let’s take a look at answer choice (C). This rule is consistent with our original diagram, in which—as discussed above—F must be released either first or second. However, this rule by itself does not require F to be released before both J and L. If F were released second, for instance, it would be possible that L is released first, which would violate the rule being replaced. Therefore, answer choice (C) contains a Partial Match, and must be eliminated.
Answer choice (A): This is the correct answer choice. If only K can be released earlier than F, it follows that F must be released earlier than all other films, with the exception of K. This stipulation is consistent with the original diagram, because the rule we are asked to replace forced F into one of the first two slots (before J, H, L, and G). Answer choice (A) does not therefore contain an Additional Effects rule. Answer choice (A) is not a Partial Match either: if F is released earlier than all other films except for K, it logically follows that F must be released earlier than J and L. Thus, the effect is identical, and this is the correct answer choice.
(The complete setup for this game can be found here: lsat/viewtopic.php?t=14958)
The correct answer choice is (A)
This is the first of two Rule Substitution questions on the December 2013 LSAT. The question stem requires you to replace the first rule of the game, which created the following relationship:
The rule itself is relatively straightforward. Without it, we are left with the following sequencing relationships:
- K J H
L G
Since Additional Effects are easier to spot, look for answer choices that contain such conditions first. You should immediately notice, for instance, that answer choices (B) and (D) add conditions that are extraneous to our original rule set: K need not be released earlier than L, and F does not have to be released earlier than K. Both of these answer choices should quickly be eliminated.
Among the Partial Match answer choices, (E) is the easiest one to eliminate. While the rule we are asked to substitute requires either F or K to be released first, this inference—by itself—does not have the exact same effect on the order in which the films are released. For instance, if K were released first, the relationship between F, J, and L would remain completely uncertain. Because answer choice (E) can limit K without limiting F in any manner, it is incorrect.
Next, let’s take a look at answer choice (C). This rule is consistent with our original diagram, in which—as discussed above—F must be released either first or second. However, this rule by itself does not require F to be released before both J and L. If F were released second, for instance, it would be possible that L is released first, which would violate the rule being replaced. Therefore, answer choice (C) contains a Partial Match, and must be eliminated.
Answer choice (A): This is the correct answer choice. If only K can be released earlier than F, it follows that F must be released earlier than all other films, with the exception of K. This stipulation is consistent with the original diagram, because the rule we are asked to replace forced F into one of the first two slots (before J, H, L, and G). Answer choice (A) does not therefore contain an Additional Effects rule. Answer choice (A) is not a Partial Match either: if F is released earlier than all other films except for K, it logically follows that F must be released earlier than J and L. Thus, the effect is identical, and this is the correct answer choice.
You do not have the required permissions to view the files attached to this post.